You are on page 1of 6

Math 104, Solution to Homework 2

Instructor: Guoliang Wu

June 21, 2009

Ross, K. A., Elementary Analysis: The theory of calculus:


Page 18: 3.3, 3.4, 3.6, 3.7, 3.8
Page 25: 4.1 – 4.4 (only (b), (j), (n), (v)), 4.6, 4.7, 4.11, 4.15

3.3 Prove that (iv) (−a)(−b) = ab for all a, b; (v) ac = bc and c 6= 0


imply a = b.

Proof. (iv) Using result (iii) in Theorem 3.1 (Ross), we have

(−a)(−b) = −a(−b) = −(−b)a = −(−ba) = ba = ab.

(v) ac = bc ⇒ ac + (−bc) = bc + (−bc) = 0 ⇒ ac + (−b)c = 0 (by (iii)).


Then by DL and M2, we obtain c(a + (−b)) = 0. Since c 6= 0, c−1
exists, and by M4 and (ii),

a + (−b) = c−1 c(a + (−b)) = c−1 · 0 = 0.

Finally, add b to both sides and use A1, a + ((−b) + b) = 0 + b ⇒


a=b

3.4 Prove that (v) 0 < 1; (vii) if 0 < a < b, then 0 < b−1 < a−1 .

Proof. (v) 1 = 1 · 1, so 0 ≤ 1 by (iv) of Theorem 3.2 (Ross). How-


ever, 0 6= 1, so 0 < 1.
(vii) 0 < a implies that 0 < a−1 by (vi). Also 0 < a < b implies
0 < b hence 0 < b−1 again by (vi). Starting from a < b, we have
a · a−1 < b · a−1 and then (a · a−1 ) · b−1 < (b · a−1 ) · b−1 ⇒ b−1 <
b · (a−1 · b−1 ) = b · (b−1 · a−1 ) = (b · b−1 ) · a−1 = 1 · a−1 = a−1 . So
0 < b−1 < a−1 .

3.6 (a) Prove that |a + b + c| ≤ |a| + |b| + |c| for all a, b, c ∈ R.

1
(b) Use induction to prove

|a1 + a2 + · · · + an | ≤ |a1 | + |a2 | + · · · + |an |

for n numbers a1 , a2 , · · · , an .

Proof. (a) Using Triangle inequality twice,

|a + b + c| = |(a + b) + c| ≤ |a + b| + |c| ≤ |a| + |b| + |c|.

(b) (1) When n = 1, the inequality holds since |a1 | ≤ |a1 |.


(2) Suppose the inequality holds for n numbers, i.e.,

|a1 + a2 + · · · + an | ≤ |a1 | + |a2 | + · · · + |an |.

We want to show that

|a1 + a2 + · · · + an + an+1 | ≤ |a1 | + |a2 | + · · · + |an | + |an+1 |.

By Triangle Inequality and the above assumption,

|a1 + a2 + · · · + an + an+1 | = |(a1 + a2 + · · · + an ) + an+1 |


≤||a1 + a2 + · · · + an | + |an+1 |
≤|a1 | + |a2 | + · · · + |an | + |an+1 |.

By induction, we proved the claim.

3.7 (a) Show that |b| < a if and only if −a < b < a.
(b) Show that |a − b| < c if and only if b − c < a < b + c.
(c) Show that |a − b| ≤ c if and only if b − c ≤ a ≤ b + c.
To prove a statement like “A if and only if B” (or, “A and B
are equivalent”), we need to show that (1) A implies B, and
(2) B implies A.

Proof. It is clear that

−|x| ≤ x ≤ |x|

for any real number x since |x| is either x, or −x. (You can use
this fact later on.)

2
(a) “⇒”: From the above inequality, −|b| ≤ b ≤ |b|. So b ≤ |b| <
a which implies b < a by O3. (b = a cannot happen since
otherwise, |b| = a.) On the other hand, −|b| ≤ b ⇒ −b ≤
|b| < a. Thus −b < a. So −a < b. We proved that

−a < b < a.

“⇐”: If b ≥ 0, then |b| = b. Thus |b| < a since b < a. If b < 0,


then |b| = −b < a since −a < b.
(b) By what we have proved in (a),

|a − b| < c if and only if − c < a − b < c.

Since −c < a − b < c if and only if b − c < a < b + c, we have

|a − b| < c if and only if b − c < a < b + c.

(c) The same argument as in (a) shows that |b| ≤ a if and only
if −a ≤ b ≤ a. (This is Exercise 3.5 (a).)
“⇒”: Again, we use the obvious fact that −|b| ≤ b ≤ |b|. So
b ≤ |b| ≤ a which implies b ≤ a by O3. On the other hand,
−|b| ≤ b ⇒ −b ≤ |b| ≤ a. Thus −b ≤ a. So −a ≤ b. We proved
that
−a ≤ b ≤ a.
“⇐”: If b ≥ 0, then |b| = b. Thus |b| ≤ a since b ≤ a. If b < 0,
then |b| = −b ≤ a since −a ≤ b.
Therefore,

|a − b| ≤ c if and only if − c ≤ a − b ≤ c.

Since −c ≤ a − b ≤ c if and only if b − c ≤ a ≤ b + c, we have

|a − b| ≤ c if and only if b − c ≤ a ≤ b + c.

3.8 Let a, b ∈ R. Show that if a ≤ b1 for every b1 > b, then a ≤ b.

Proof. We prove it by contradiction. Suppose b < a, then we


take
1
b1 = (a + b).
2
We check that b1 > b since a + b > 2b. However, b1 < a since
a+b < 2a. We obtain a contradiction. This proves that a ≤ b.

3
4.1 – 4.4 (b) (0, 1) is bounded above: it has lots of upper bounds. For
instance, 1, 2, 3. It has lower bounds 0, −1, −2 (or any non-
positive number.)

sup(0, 1) = 1, inf(0, 1) = 0.

(j) {1 − 31n : n ∈ N} = { 23 , 89 , 27
26
, · · · }. Upper bounds: 1, 2, 3; lower
2
bounds: 3 , 0, −1.
1 1 2
sup{1 − : n ∈ N} = 1, inf{1 − : n ∈ N} = .
3n 3n 3
(n) {r ∈ Q : r < 2}. Upper bounds: 2, 3, 4. The set is NOT
bounded below.

sup{r ∈ Q : r < 2} = 2.

(We will learn the notation −∞. This set is not bounded
below, so it does not have a lower bound/infimum. But
sometimes we write inf{r ∈ Q : r < 2} = −∞ to mean that
it is not bound below.)
(v) {cos nπ

3 : n ∈ N}. Note that this is in fact a finite set since
cos is a cyclic function:
 nπ  1 1
{cos : n ∈ N} = { , − , −1, 1},
3 2 2
since cos π3 = 12 , cos 2π = − 12 , cos 3π = −1, cos 4π
   
3 3 3 =
− 21 , cos 5π3 = 1
2 , cos 6π
3 = 1, · · ·
Upper bounds: 1, 2, 3; lower bounds −1, −2, −3.
 nπ   nπ 
sup{cos : n ∈ N} = 1, inf{cos : n ∈ N} = −1.
3 3

4.6 Let S be a nonempty bounded subset of R.

(a) Prove that inf S ≤ sup S.


(b) What can you say about S if inf S = sup S.

Proof. (a) Take any element s ∈ S (nonempty). Note that sup S is


an upper bound of S and inf S is a lower bound, thus s ≤ sup S
and inf S ≤ s. Therefore

inf S ≤ sup S.

4
(b) If inf S = sup S, then the set S has only one element inf S
(same as sup S). This is because, as argued in (a), for any ele-
ment s ∈ S,
inf S ≤ s ≤ sup S = inf S.
Then it must be that

inf S = s = sup S = inf S.

Otherwise, there is a contradiction inf S < inf S.


Any element s equals inf S means that the set S has only one
element.

4.7 Let S and T be nonempty bounded subsets of R.

(a) Prove that if S ⊂ T , then inf T ≤ inf S ≤ sup S ≤ sup T .


(b) Prove that sup(S ∪ T ) = max{sup S, sup T }.

Proof. (a) In 4.6 (a), we proved that inf S ≤ sup S for any subset
S of R. So we need to prove that inf T ≤ inf S and sup S ≤ sup T .
To show that inf T ≤ inf S, it suffices to show that inf T is a
lower bound of S, then it follows that inf T ≤ inf S since inf S
is the greatest lower bound (greater than or equal to any other
lower bound of S).
For any s ∈ S, it belongs to T since S ⊂ T . Thus inf T ≤ s since
inf T is a lower bound of T and s ∈ T . Then, by definition, inf T
is a lower bound of S and therefore inf T ≤ inf S.
Similarly, to show sup S ≤ sup T , we need only to show that
sup T is an upper bound of S. For any s ∈ S, it is also in T
because S ⊂ T . Thus s ≤ sup T . Therefore sup T is an upper
bound of S.
(b) Note that S ⊂ S ∪ T and T ⊂ S ∪ T . We have

sup S ≤ sup(S ∪ T ); sup T ≤ sup(S ∪ T ).

So
max{sup S, sup T } ≤ sup(S ∪ T ).

On the other hand, for any element s ∈ S ∪ T , either (i) s ∈ S or


(ii) s ∈ T . If (i) happens (s ∈ S), then s ≤ sup S ≤ max{sup S, sup T }.

5
Otherwise (ii) happens (s ∈ T ), which implies that s ≤ sup T ≤
max{sup S, sup T }. In either case

s ≤ max{sup S, sup T }.

Since s is taken arbitrarily from S∪T , we see that max{sup S, sup T }


is an upper bound of S ∪ T . Thus

sup(S ∪ T ) ≤ max{sup S, sup T }.

Finally, we conclude that

sup(S ∪ T ) = max{sup S, sup T }.

4.11 Consider a, b ∈ R where a < b. Use denseness of Q to show that


there are infinitely many rationals between a and b.

Proof. See textbook (P313).

1
4.15 Let a, b ∈ R. Show that if a ≤ b + n for all n ∈ N, then a ≤ b.

Proof. Suppose not, i.e., b < a. Then a − b > 0. By Archimedean


Property, there exists an integer n such that n(a − b) > 1, which
implies that a − b > n1 and a > b + n1 . This is a contradiction to
what we have: a ≤ b + n1 for all n ∈ N. Therefore a ≤ b.

You might also like